Diễn Đàn MathScopeDiễn Đàn MathScope
  Diễn Đàn MathScope
Ghi Danh Hỏi/Ðáp Thành Viên Social Groups Lịch Ðánh Dấu Ðã Ðọc

Go Back   Diễn Đàn MathScope > Sơ Cấp > Việt Nam và IMO > 2012

News & Announcements

Ngoài một số quy định đã được nêu trong phần Quy định của Ghi Danh , mọi người tranh thủ bỏ ra 5 phút để đọc thêm một số Quy định sau để khỏi bị treo nick ở MathScope nhé !

* Nội quy MathScope.Org

* Một số quy định chung !

* Quy định về việc viết bài trong diễn đàn MathScope

* Nếu bạn muốn gia nhập đội ngũ BQT thì vui lòng tham gia tại đây

* Những câu hỏi thường gặp

* Về việc viết bài trong Box Đại học và Sau đại học


Trả lời Gởi Ðề Tài Mới
 
Ðiều Chỉnh Xếp Bài
Old 09-07-2012, 09:49 PM   #1
novae
+Thành Viên Danh Dự+
 
novae's Avatar
 
Tham gia ngày: Jul 2010
Đến từ: Event horizon
Bài gởi: 2,453
Thanks: 53
Thanked 3,057 Times in 1,288 Posts
[IMO 2012] Bài 2 - Bất đẳng thức

Cho số nguyên $n \ge 3$ và các số thực dương $a_2,a_3,\ldots,a_n$ thỏa mãn $a_2 \cdots a_n= 1$. Chứng minh rằng
$$ (1+a_2)^2(1+a_3)^3 \cdots (1+a_n)^n > n^n $$
[RIGHT][I][B]Nguồn: MathScope.ORG[/B][/I][/RIGHT]
 
__________________
M.

thay đổi nội dung bởi: novae, 11-07-2012 lúc 12:41 AM
novae is offline   Trả Lời Với Trích Dẫn
The Following 4 Users Say Thank You to novae For This Useful Post:
boykhtna1 (11-07-2012), pco (11-07-2012), Shuichi Akai (11-07-2012), yamatunga (11-07-2012)
Old 11-07-2012, 01:36 AM   #2
Harry Potter
+Thành Viên+
 
Harry Potter's Avatar
 
Tham gia ngày: Sep 2008
Bài gởi: 32
Thanks: 24
Thanked 26 Times in 6 Posts
Dùng cái này là xong mà $(a_k+1)=\left(a_k+\frac 1{k-1}+\cdots+\frac 1{k-1}\right)\geq k\sqrt[k]{\frac{a_k}{(k-1)^{k-1}}}$
[RIGHT][I][B]Nguồn: MathScope.ORG[/B][/I][/RIGHT]
 
Harry Potter is offline   Trả Lời Với Trích Dẫn
The Following 5 Users Say Thank You to Harry Potter For This Useful Post:
baotram (11-07-2012), hoang_kkk (13-07-2012), kien10a1 (11-07-2012), nghiepdu-socap (11-07-2012), ngocson_dhsp (11-07-2012)
Old 11-07-2012, 01:56 AM   #3
Shuichi Akai
+Thành Viên+
 
Tham gia ngày: Nov 2011
Bài gởi: 12
Thanks: 40
Thanked 10 Times in 8 Posts
Mình làm giống như ý tưởng bạn ở trên
AM - GM cho k số:
$(1+a_k)^k=\left (\dfrac{1}{k-1}.(k-1)+a_k\right )^k\ge \dfrac{a_k}{(k-1)^{k-1}}.k^k $
từ đấy suy ra
$\prod (1+a_i)^i\geq a_2a_3...a_n\frac{2^2.3^3...n^n}{1^1.2^2...(n-1)^{n-1}}=n^n $
dấu bằng xảy ra $\Leftrightarrow a_i = \frac{1}{i-1} \Rightarrow a_2a_3...a_n < 1 $ (mâu thuẫn)
Vậy ta có đpcm
[RIGHT][I][B]Nguồn: MathScope.ORG[/B][/I][/RIGHT]
 
Shuichi Akai is offline   Trả Lời Với Trích Dẫn
Old 11-07-2012, 08:28 AM   #4
AnhIsGod
+Thành Viên+
 
Tham gia ngày: Nov 2011
Đến từ: Vô cực
Bài gởi: 267
Thanks: 358
Thanked 48 Times in 32 Posts
Bài này chẳng phải AM-GM trực tiếp cho từng thừa số sau khi tách ra rồi xét dấu bằng thì là gì?
[RIGHT][I][B]Nguồn: MathScope.ORG[/B][/I][/RIGHT]
 
AnhIsGod is offline   Trả Lời Với Trích Dẫn
Old 11-07-2012, 09:27 AM   #5
hamaianh0405
+Thành Viên+
 
Tham gia ngày: May 2012
Bài gởi: 107
Thanks: 59
Thanked 7 Times in 6 Posts
Vậy nó cho cái tích $a_{1}a_{2}...a_{n}=1 $ để làm gì vì dấu bằng mâu thuẫn
[RIGHT][I][B]Nguồn: MathScope.ORG[/B][/I][/RIGHT]
 
hamaianh0405 is offline   Trả Lời Với Trích Dẫn
The Following User Says Thank You to hamaianh0405 For This Useful Post:
AnhIsGod (11-07-2012)
Old 11-07-2012, 10:34 AM   #6
Toan95cqb
+Thành Viên+
 
Tham gia ngày: Jun 2011
Bài gởi: 59
Thanks: 17
Thanked 19 Times in 13 Posts
Trích:
Nguyên văn bởi hamaianh0405 View Post
Vậy nó cho cái tích $a_{1}a_{2}...a_{n}=1 $ để làm gì vì dấu bằng mâu thuẫn
Bài này không có dấu bằng. Bạn xem lại đề đi $>n^n $
[RIGHT][I][B]Nguồn: MathScope.ORG[/B][/I][/RIGHT]
 
Toan95cqb is offline   Trả Lời Với Trích Dẫn
Old 11-07-2012, 08:31 PM   #7
hansongkyung
+Thành Viên+
 
Tham gia ngày: Jan 2012
Đến từ: Han Tae Woong - IMO 1998
Bài gởi: 493
Thanks: 109
Thanked 417 Times in 241 Posts
Gửi tin nhắn qua Yahoo chát tới hansongkyung
Một cách khác dựa trên ý tưởng trên
Vì $a_2a_3...a_n =1$ nên tồn tại các số $x_2,x_3,...,x_n$ sao cho:
$a_2=\frac{x_2}{x_3}, a_3=\frac{x_4}{x_3},...,a_n=\frac{x_n}{x_2}$
Sử dụng bđt AM-GM ta có
$(\frac{x_i+x_{i+1}}{x_{i+1}})^k \ge \frac{i^{i}}{(i-1)^{i-1}
}\frac{x_{i}}{x_{i+1}}$
Cho $i$ chạy từ $2 \to n$ ta có ĐPCM

Việt Nam cố lên
[RIGHT][I][B]Nguồn: MathScope.ORG[/B][/I][/RIGHT]
 
hansongkyung is offline   Trả Lời Với Trích Dẫn
The Following 3 Users Say Thank You to hansongkyung For This Useful Post:
DuyLTV (11-07-2012), lexuanthang (13-07-2012), nguoibimat (12-07-2012)
Old 12-07-2012, 09:55 PM   #8
minhphuc.v
+Thành Viên+
 
Tham gia ngày: Jul 2012
Bài gởi: 5
Thanks: 1
Thanked 2 Times in 2 Posts
Dùng BĐT Holder ta có
$$(1+1)^1(1+a_2)^2(1+a_3)^3...(1+a_n)^n \ge (1+a_2.a_3...a_n)^{\frac{n(n+1)}{2}}$$
$$=2^{\frac{n(n+1)}{2}} = \left( 2^{\frac{(n+1)}{2}}\right)^n \ge ( 2\ln2.n )^n=n^n(2\ln2)^n > 2n^n, (n \ge 3)$$
Vì ta có hàm số $$f(x)=2^{\frac{x+1}{2}}-2\ln2.x \ge 0, \forall x \ge 3$$
[RIGHT][I][B]Nguồn: MathScope.ORG[/B][/I][/RIGHT]
 

thay đổi nội dung bởi: minhphuc.v, 12-07-2012 lúc 10:05 PM
minhphuc.v is offline   Trả Lời Với Trích Dẫn
The Following User Says Thank You to minhphuc.v For This Useful Post:
9A1 (13-07-2012)
Trả lời Gởi Ðề Tài Mới

Bookmarks

Ðiều Chỉnh
Xếp Bài

Quuyền Hạn Của Bạn
You may not post new threads
You may not post replies
You may not post attachments
You may not edit your posts

BB code is Mở
Smilies đang Mở
[IMG] đang Mở
HTML đang Tắt

Chuyển đến


Múi giờ GMT. Hiện tại là 03:21 PM.


Powered by: vBulletin Copyright ©2000-2024, Jelsoft Enterprises Ltd.
Inactive Reminders By mathscope.org
[page compression: 65.83 k/74.96 k (12.18%)]